Re: [obm-l] Teoria dos numeros

2020-05-22 Por tôpico Anderson Torres
Em ter., 19 de mai. de 2020 às 15:52, Israel Meireles Chrisostomo
 escreveu:
>
> Olá pessoal.Ultimamente tenho pensado em como provar que a tangente de um 
> arco racional diferente de zero é sempre irracional.

Cê diz que se r é racional então tan(r) é irracional (exceto se r=0)?

Acho que dá para ser mais arrojado e provar logo a transcendência.
Afinal, qualquer racional pode ser multiplicado e dividido até dar 1,
e as funções tangente de múltiplo arco são racionais no arco primeiro,
e se tan 1 é transcedente, acabou.

Eu consegui chegar no seguinte: Se r é real diferente  de zero e s é
inteiro diferente de zero, então ou tan(r-1/2s)  ou tan(r) é
irracional.
> Daí então eu tomo um r racional,  então ou tan(r-1/2s)  ou tan(r) é 
> irracional, se tan(r) é irracional então está provado, se por um outro lado 
> tan(r-1/2s)  é irracional então faça r= r'+1/2s e daí tem-se tan(r') é 
> irracional.O que mostra que a tangente de todo arco racional diferente de 
> zero é irracional.
> Está correto esse meu raciocínio?
> Partindo de que "se r é real diferente  de zero e s é inteiro diferente de 
> zero, então ou tan(r-1/2s)  ou tan(r) é irracional "  como posso provar isso ?
> --
> Israel Meireles Chrisostomo
>
> --
> Esta mensagem foi verificada pelo sistema de antivírus e
> acredita-se estar livre de perigo.

-- 
Esta mensagem foi verificada pelo sistema de antiv�rus e
 acredita-se estar livre de perigo.


=
Instru��es para entrar na lista, sair da lista e usar a lista em
http://www.mat.puc-rio.br/~obmlistas/obm-l.html
=


Re: [obm-l] teoria dos numeros

2020-03-30 Por tôpico Carlos Victor
 

Basta fazer (2^3-1)^2n+(2^3+1)^2n -2 e usar binômio de Newton. 

Em 28/03/2020 13:55, Israel Meireles Chrisostomo escreveu: 

> Eu sei resolver o problema abaixo,porém não sei se é a forma mais simples de 
> se fazer.Vcs poderiam por favor colocar suas soluções nos comentários dessa 
> publicação? O problema é o seguinte:
> Prove que 128 divide 49^{n} + 81^{n} -2, para todo n ≥ 1.Se possível não use 
> indução, pois eu já estou usando indução. 
> 
> -- 
> 
> Israel Meireles Chrisostomo 
> -- 
> Esta mensagem foi verificada pelo sistema de antivrus e 
> acredita-se estar livre de perigo.

 
-- 
Esta mensagem foi verificada pelo sistema de antiv�rus e
 acredita-se estar livre de perigo.



Re: [obm-l] teoria dos numeros

2020-03-30 Por tôpico Pedro José
Boa noite!

errata:
Ao invés de: 128=2^7 então 2^7| 49^{n} + 81^{n} −2<==> x= 2^7| 49^{n} +
81^{n}=2 mod2^7
128=2^7 então 2^7| 49^{n} + 81^{n} −2<==> x= 49^{n} + 81^{n}=2 mod2^7

Saudações,
PJMS

Em dom., 29 de mar. de 2020 às 14:04, Pedro José 
escreveu:

> Bom dia!
> Prove que 128 divide 49^{n} + 81^{n} −2, para todo n ≥ 1.
> 128=2^7 então 2^7| 49^{n} + 81^{n} −2<==> x= 2^7| 49^{n} + 81^{n}=2 mod2^7
> x= a + b , a= 49^n e b=81^n
> a= (64-15)^n = n(-1)^n*n*64*(15)^(n-1) + (-1)^n*15^n mod2^7; pois, os
> demais termos do binômio de Newton terão o fator (2^6)^m com m>1 que é
> côngruo 0 mod2^7.
> b= (64+17)^n = n*64*17^(n-1) + 17^n mod2^7 pelo mesmo motivo anterior.
> a+b = n*64(17^n-1 +(-1)^(n-1)*15^(n-1)) + 17^n + (-1)^n*15^n =
> (-1)^(n-1)*15^(n-1)) + 17^n + (-1)^n*15^n mod2^; pois a primeira parcela é
> côngrua a 0 mod2^7; já que o termo entre parêntesis é par.
> (16+1)^n= n*16+1 mod2^7 ,pois, (2^4)^m =0 mod2^7 para m>1
> (-1)^n*(16-1)= (-1)^n*[(-1)^(n-1)*n*16+(-1)^n]=-16n +1
> então x = a+b= 2 mod2^7 ==> 2^7 | a+b-2
>
> Saudações,
> PJMS
>
>
>
>
> Em sáb., 28 de mar. de 2020 às 14:05, Israel Meireles Chrisostomo <
> israelmchrisost...@gmail.com> escreveu:
>
>> Eu sei resolver o problema abaixo,porém não sei se é a forma mais simples
>> de se fazer.Vcs poderiam por favor colocar suas soluções nos comentários
>> dessa publicação? O problema é o seguinte:
>> Prove que 128 divide 49^{n} + 81^{n} −2, para todo n ≥ 1.Se possível não
>> use indução, pois eu já estou usando indução.
>>
>> --
>> Israel Meireles Chrisostomo
>>
>> --
>> Esta mensagem foi verificada pelo sistema de antivírus e
>> acredita-se estar livre de perigo.
>
>

-- 
Esta mensagem foi verificada pelo sistema de antiv�rus e
 acredita-se estar livre de perigo.



Re: [obm-l] teoria dos numeros

2020-03-29 Por tôpico Pedro José
Bom dia!
Prove que 128 divide 49^{n} + 81^{n} −2, para todo n ≥ 1.
128=2^7 então 2^7| 49^{n} + 81^{n} −2<==> x= 2^7| 49^{n} + 81^{n}=2 mod2^7
x= a + b , a= 49^n e b=81^n
a= (64-15)^n = n(-1)^n*n*64*(15)^(n-1) + (-1)^n*15^n mod2^7; pois, os
demais termos do binômio de Newton terão o fator (2^6)^m com m>1 que é
côngruo 0 mod2^7.
b= (64+17)^n = n*64*17^(n-1) + 17^n mod2^7 pelo mesmo motivo anterior.
a+b = n*64(17^n-1 +(-1)^(n-1)*15^(n-1)) + 17^n + (-1)^n*15^n =
(-1)^(n-1)*15^(n-1)) + 17^n + (-1)^n*15^n mod2^; pois a primeira parcela é
côngrua a 0 mod2^7; já que o termo entre parêntesis é par.
(16+1)^n= n*16+1 mod2^7 ,pois, (2^4)^m =0 mod2^7 para m>1
(-1)^n*(16-1)= (-1)^n*[(-1)^(n-1)*n*16+(-1)^n]=-16n +1
então x = a+b= 2 mod2^7 ==> 2^7 | a+b-2

Saudações,
PJMS




Em sáb., 28 de mar. de 2020 às 14:05, Israel Meireles Chrisostomo <
israelmchrisost...@gmail.com> escreveu:

> Eu sei resolver o problema abaixo,porém não sei se é a forma mais simples
> de se fazer.Vcs poderiam por favor colocar suas soluções nos comentários
> dessa publicação? O problema é o seguinte:
> Prove que 128 divide 49^{n} + 81^{n} −2, para todo n ≥ 1.Se possível não
> use indução, pois eu já estou usando indução.
>
> --
> Israel Meireles Chrisostomo
>
> --
> Esta mensagem foi verificada pelo sistema de antivírus e
> acredita-se estar livre de perigo.

-- 
Esta mensagem foi verificada pelo sistema de antiv�rus e
 acredita-se estar livre de perigo.



Re: [obm-l] Teoria dos numeros

2018-03-28 Por tôpico Pedro José
Bom dia!

Não deu para compreender. Para cada terno (k,j,w) terá apenas uma raiz em x
ou nenhuma. Mas para todo natural existe pelo menos um terno que atenda a
sua proposição.

w=x ; k=1 e j=2.

Saudações,
PJMS

Em 27 de março de 2018 22:28, Israel Meireles Chrisostomo <
israelmchrisost...@gmail.com> escreveu:

> O máximo que eu consigo é considerar uma solução que seja um número primo
>
> Em 27 de março de 2018 22:27, Israel Meireles Chrisostomo <
> israelmchrisost...@gmail.com> escreveu:
>
>> Está muito geral essas condições, achei que pudesse conseguir alguma
>> restrição a fim de resolver um outro problema, mas talvez esse caminho não
>> é muito apropriado
>>
>> Em 27 de março de 2018 22:10, Claudio Buffara 
>> escreveu:
>>
>>> O problema é só esse mesmo?
>>> Não tem nenhum contexto? Não é dada nenhuma relação entre k, j e w?
>>>
>>>
>>> 2018-03-27 21:27 GMT-03:00 Anderson Torres >> >:
>>>
 Em 27 de março de 2018 21:06, Israel Meireles Chrisostomo
  escreveu:
 > Ola pessoal eu gostaria de saber quantas são e quais são as soluções
 > naturais de (x+w)k=xj  na variável x, onde k e j  e w são naturais
 dados
 >

 (x+w)k=xj
 xk+wk=xj

 wk=xj-xk
 wk=x(j-k)

 x=wk/(j-k)


 > --
 > Israel Meireles Chrisostomo
 >
 > --
 > Esta mensagem foi verificada pelo sistema de antivírus e
 > acredita-se estar livre de perigo.

 --
 Esta mensagem foi verificada pelo sistema de antivírus e
  acredita-se estar livre de perigo.


 
 =
 Instru�ões para entrar na lista, sair da lista e usar a lista em
 http://www.mat.puc-rio.br/~obmlistas/obm-l.html
 
 =

>>>
>>>
>>> --
>>> Esta mensagem foi verificada pelo sistema de antivírus e
>>> acredita-se estar livre de perigo.
>>>
>>
>>
>>
>> --
>> Israel Meireles Chrisostomo
>>
>
>
>
> --
> Israel Meireles Chrisostomo
>
> --
> Esta mensagem foi verificada pelo sistema de antivírus e
> acredita-se estar livre de perigo.
>

-- 
Esta mensagem foi verificada pelo sistema de antiv�rus e
 acredita-se estar livre de perigo.



Re: [obm-l] Teoria dos numeros

2018-03-27 Por tôpico Israel Meireles Chrisostomo
Está muito geral essas condições, achei que pudesse conseguir alguma
restrição a fim de resolver um outro problema, mas talvez esse caminho não
é muito apropriado

Em 27 de março de 2018 22:10, Claudio Buffara 
escreveu:

> O problema é só esse mesmo?
> Não tem nenhum contexto? Não é dada nenhuma relação entre k, j e w?
>
>
> 2018-03-27 21:27 GMT-03:00 Anderson Torres :
>
>> Em 27 de março de 2018 21:06, Israel Meireles Chrisostomo
>>  escreveu:
>> > Ola pessoal eu gostaria de saber quantas são e quais são as soluções
>> > naturais de (x+w)k=xj  na variável x, onde k e j  e w são naturais dados
>> >
>>
>> (x+w)k=xj
>> xk+wk=xj
>>
>> wk=xj-xk
>> wk=x(j-k)
>>
>> x=wk/(j-k)
>>
>>
>> > --
>> > Israel Meireles Chrisostomo
>> >
>> > --
>> > Esta mensagem foi verificada pelo sistema de antivírus e
>> > acredita-se estar livre de perigo.
>>
>> --
>> Esta mensagem foi verificada pelo sistema de antivírus e
>>  acredita-se estar livre de perigo.
>>
>>
>> =
>> Instru�ões para entrar na lista, sair da lista e usar a lista em
>> http://www.mat.puc-rio.br/~obmlistas/obm-l.html
>> =
>>
>
>
> --
> Esta mensagem foi verificada pelo sistema de antivírus e
> acredita-se estar livre de perigo.
>



-- 
Israel Meireles Chrisostomo

-- 
Esta mensagem foi verificada pelo sistema de antiv�rus e
 acredita-se estar livre de perigo.



Re: [obm-l] Teoria dos numeros

2018-03-27 Por tôpico Israel Meireles Chrisostomo
O máximo que eu consigo é considerar uma solução que seja um número primo

Em 27 de março de 2018 22:27, Israel Meireles Chrisostomo <
israelmchrisost...@gmail.com> escreveu:

> Está muito geral essas condições, achei que pudesse conseguir alguma
> restrição a fim de resolver um outro problema, mas talvez esse caminho não
> é muito apropriado
>
> Em 27 de março de 2018 22:10, Claudio Buffara 
> escreveu:
>
>> O problema é só esse mesmo?
>> Não tem nenhum contexto? Não é dada nenhuma relação entre k, j e w?
>>
>>
>> 2018-03-27 21:27 GMT-03:00 Anderson Torres 
>> :
>>
>>> Em 27 de março de 2018 21:06, Israel Meireles Chrisostomo
>>>  escreveu:
>>> > Ola pessoal eu gostaria de saber quantas são e quais são as soluções
>>> > naturais de (x+w)k=xj  na variável x, onde k e j  e w são naturais
>>> dados
>>> >
>>>
>>> (x+w)k=xj
>>> xk+wk=xj
>>>
>>> wk=xj-xk
>>> wk=x(j-k)
>>>
>>> x=wk/(j-k)
>>>
>>>
>>> > --
>>> > Israel Meireles Chrisostomo
>>> >
>>> > --
>>> > Esta mensagem foi verificada pelo sistema de antivírus e
>>> > acredita-se estar livre de perigo.
>>>
>>> --
>>> Esta mensagem foi verificada pelo sistema de antivírus e
>>>  acredita-se estar livre de perigo.
>>>
>>>
>>> 
>>> =
>>> Instru�ões para entrar na lista, sair da lista e usar a lista em
>>> http://www.mat.puc-rio.br/~obmlistas/obm-l.html
>>> 
>>> =
>>>
>>
>>
>> --
>> Esta mensagem foi verificada pelo sistema de antivírus e
>> acredita-se estar livre de perigo.
>>
>
>
>
> --
> Israel Meireles Chrisostomo
>



-- 
Israel Meireles Chrisostomo

-- 
Esta mensagem foi verificada pelo sistema de antiv�rus e
 acredita-se estar livre de perigo.



Re: [obm-l] Teoria dos numeros

2018-03-27 Por tôpico Claudio Buffara
O problema é só esse mesmo?
Não tem nenhum contexto? Não é dada nenhuma relação entre k, j e w?


2018-03-27 21:27 GMT-03:00 Anderson Torres :

> Em 27 de março de 2018 21:06, Israel Meireles Chrisostomo
>  escreveu:
> > Ola pessoal eu gostaria de saber quantas são e quais são as soluções
> > naturais de (x+w)k=xj  na variável x, onde k e j  e w são naturais dados
> >
>
> (x+w)k=xj
> xk+wk=xj
>
> wk=xj-xk
> wk=x(j-k)
>
> x=wk/(j-k)
>
>
> > --
> > Israel Meireles Chrisostomo
> >
> > --
> > Esta mensagem foi verificada pelo sistema de antivírus e
> > acredita-se estar livre de perigo.
>
> --
> Esta mensagem foi verificada pelo sistema de antivírus e
>  acredita-se estar livre de perigo.
>
>
> =
> Instru�ões para entrar na lista, sair da lista e usar a lista em
> http://www.mat.puc-rio.br/~obmlistas/obm-l.html
> =
>

-- 
Esta mensagem foi verificada pelo sistema de antiv�rus e
 acredita-se estar livre de perigo.



Re: [obm-l] Teoria dos numeros

2018-03-27 Por tôpico Anderson Torres
Em 27 de março de 2018 21:06, Israel Meireles Chrisostomo
 escreveu:
> Ola pessoal eu gostaria de saber quantas são e quais são as soluções
> naturais de (x+w)k=xj  na variável x, onde k e j  e w são naturais dados
>

(x+w)k=xj
xk+wk=xj

wk=xj-xk
wk=x(j-k)

x=wk/(j-k)


> --
> Israel Meireles Chrisostomo
>
> --
> Esta mensagem foi verificada pelo sistema de antivírus e
> acredita-se estar livre de perigo.

-- 
Esta mensagem foi verificada pelo sistema de antiv�rus e
 acredita-se estar livre de perigo.


=
Instru��es para entrar na lista, sair da lista e usar a lista em
http://www.mat.puc-rio.br/~obmlistas/obm-l.html
=


Re: [obm-l] Teoria dos numeros

2016-12-22 Por tôpico Pedro José
Bom dia!

x= 0  y= 1 e z= 1 ; a = -1, b=-1 e c=-1

-1.0 + -1.1 + -1.1 = -1 + 0 -1 (V) atende a

1 + 1 =1 > = 0 +1 +1 (V) atende b.

-1 não é soma de três quadrados de inteiros.

Tem que ter mais restrições.

Saudações,
PJMS



Em 20 de dezembro de 2016 19:08, Gabriel Tostes 
escreveu:

> A,b,c,X,y,z inteiros tais que
> a) ax^2+by^2+cz^2=abc +2xyz - 1
> B) ab+bc+ca>=x^2+y^2+z^2
>
> Provar que a,b,c são somas de 3 quadrados de inteiros
>
>
>
> --
> Esta mensagem foi verificada pelo sistema de antivírus e
>  acredita-se estar livre de perigo.
>
>
> =
> Instruções para entrar na lista, sair da lista e usar a lista em
> http://www.mat.puc-rio.br/~obmlistas/obm-l.html
> =
>

-- 
Esta mensagem foi verificada pelo sistema de antiv�rus e
 acredita-se estar livre de perigo.



Re: [obm-l] Teoria dos numeros

2013-11-19 Por tôpico terence thirteen
Ivan Niven and Herbert Zuckermann.

Fala de tudo que é possível ser dito de forma elementar, e é uma leitura
divertida por si mesma.


Em 28 de outubro de 2013 20:08, Hermann  escreveu:

>  Veja na livraria da SBM tem uns muito bons
>
> - Original Message -
> *From:* sergio marinho 
> *To:* obm-l@mat.puc-rio.br
> *Sent:* Monday, October 28, 2013 4:54 PM
> *Subject:* Re: [obm-l] Teoria dos numeros
>
>
> Vc poderia me indicar excelentes livros de Teoria dos números e Análise
> combinatória?
>
> Grato.  Sérgio Soares.
>
>
>  Em Sábado, 3 de Agosto de 2013 16:47, marcone augusto araújo borges <
> marconeborge...@hotmail.com> escreveu:
>   Seja n uma soma de dois numeros triangulares (a^2 + a)/2 e (b^2 + b)/2.
>   Mostre que 4n + 1 é uma soma de dois quadrados em termos de a e
> b.
>
> --
> Esta mensagem foi verificada pelo sistema de antivírus e
> acredita-se estar livre de perigo.
>
>
>
> --
> Esta mensagem foi verificada pelo sistema de antivírus e
> acredita-se estar livre de perigo.
>
>
> --
> Esta mensagem foi verificada pelo sistema de antivírus e
> acredita-se estar livre de perigo.
>



-- 
/**/
神が祝福

Torres

-- 
Esta mensagem foi verificada pelo sistema de antiv�rus e
 acredita-se estar livre de perigo.



Re: [obm-l] Teoria dos numeros

2013-10-28 Por tôpico Hermann
Veja na livraria da SBM tem uns muito bons
  - Original Message - 
  From: sergio marinho 
  To: obm-l@mat.puc-rio.br 
  Sent: Monday, October 28, 2013 4:54 PM
  Subject: Re: [obm-l] Teoria dos numeros




  Vc poderia me indicar excelentes livros de Teoria dos números e Análise 
combinatória?


  Grato.  Sérgio Soares.



  Em Sábado, 3 de Agosto de 2013 16:47, marcone augusto araújo borges 
 escreveu:

  Seja n uma soma de dois numeros triangulares (a^2 + a)/2 e (b^2 + b)/2.
   
   
   
   
   
   
   
   
   
  Mostre que 4n + 1 é uma soma de dois quadrados em termos de a e b.

  -- 
  Esta mensagem foi verificada pelo sistema de antivírus e 
  acredita-se estar livre de perigo. 




  -- 
  Esta mensagem foi verificada pelo sistema de antivírus e 
  acredita-se estar livre de perigo. 
-- 
Esta mensagem foi verificada pelo sistema de antivírus e
 acredita-se estar livre de perigo.



Re: [obm-l] Teoria dos numeros

2013-10-28 Por tôpico sergio marinho


Vc poderia me indicar excelentes livros de Teoria dos números e Análise 
combinatória?

Grato.  Sérgio Soares.




Em Sábado, 3 de Agosto de 2013 16:47, marcone augusto araújo borges 
 escreveu:
 
 
Seja n uma soma de dois numeros triangulares (a^2 + a)/2 e (b^2 + b)/2. 
Mostre que 4n + 1 é uma soma de dois quadrados em termos de a e b.
-- 
Esta mensagem foi verificada pelo sistema de antivírus e 
acredita-se estar livre de perigo. 
-- 
Esta mensagem foi verificada pelo sistema de antivírus e
 acredita-se estar livre de perigo.



Re: [obm-l] Teoria dos numeros

2013-08-03 Por tôpico Nehab

Oi, Marcone.

Ora, você quer que a soma de dois quadrados dê 2a^2 + 2b^2 + 2a + 2b + 1.
O a^2 e o b^2 saem de coisas do tipo (a + b +...)^2 e (a - b +...)^2.
Para se livrar do 2ab que aparece nessa coisas, você precisa de um +2ab 
e de um -2ab...
Dai, botando os neurônios para esquentar um pouquinho, fica fácil: (a - 
b)^2 + (a + b + 1)^2.


Abraços,
Nehab


On 03/08/2013 16:32, marcone augusto araújo borges wrote:

Seja n uma soma de dois numeros triangulares (a^2 + a)/2 e (b^2 + b)/2.
[Upload Photo to Facebook]
[Google+]
[Twitt]
[Send by Gmail]
[Upload Video to Facebook]
[Google+]
[Twitt]
[Send by Gmail]
Mostre que 4n + 1 é uma soma de dois quadrados em termos de a e b.

--
Esta mensagem foi verificada pelo sistema de antivírus e
acredita-se estar livre de perigo. 



--
Esta mensagem foi verificada pelo sistema de antivírus e
acredita-se estar livre de perigo.



Re: [obm-l] teoria dos numeros

2013-05-10 Por tôpico Bernardo Freitas Paulo da Costa
2013/5/10 terence thirteen :
> Aprenda um pouco de inglês:
>
> http://ohkawa.cc.it-hiroshima.ac.jp/www.kalva.demon.co.uk/apmo/asoln/asol972.html
>
>
> Em 10 de maio de 2013 06:48, valdir soares 
> escreveu:
>
>> Ola pessoal,
>>
>> Gostaria de saber, como fazer o problema abaixo :
>>
>> Determine n entre 100 e 1000 , tal que   ( 2+ 2^n)/n  eh tambem inteiro .

Braço por braço (a solução contando os primos e verificando que 2 é
primitiva módulo p, e depois mais braço para p, 2p, pq, 2pq), dá pra
rodar todos esses números. E (ao contrário do kalva) eu usei a
calculadora do linux em linha de comando:

$ bc
define r(n) { n ; return (2 + 2^n) % n }
for (i = 100; i <= 1000; i++) r(i)

Depois, com a resposta na mão, você apenas verifica que dá certo ;-)
-- 
Bernardo Freitas Paulo da Costa

=
Instruções para entrar na lista, sair da lista e usar a lista em
http://www.mat.puc-rio.br/~obmlistas/obm-l.html
=


Re: [obm-l] teoria dos numeros

2013-05-10 Por tôpico terence thirteen
Aprenda um pouco de inglês:

http://ohkawa.cc.it-hiroshima.ac.jp/www.kalva.demon.co.uk/apmo/asoln/asol972.html


Em 10 de maio de 2013 06:48, valdir soares escreveu:

> Ola pessoal,
>
> Gostaria de saber, como fazer o problema abaixo :
>
> Determine n entre 100 e 1000 , tal que   ( 2+ 2^n)/n  eh tambem inteiro .
>
> Obrigado
>



-- 
/**/
神が祝福

Torres


Re: [obm-l] Teoria dos numeros

2007-06-12 Por tôpico Paulo Santa Rita

Ola Carissimo Artur e demais
colegas desta lista ... OBM-L,

E facil ver que 7^4 < 10200 < 7^5. Assim, basta considerar ate 7^4. De
7 ate  10199 temos 10199 = 7 + (A-1)*7  => A = 1457 multiplos de 7.
Considerando os multiplos de 49 teriamos 10.192 = 49 + (B-1)*49 =>
B=208 multiplos de 49 e com o mesmo raciocinio achamos 29 multiplos de
343(=7^3) e 4 multiplos de 2401 (= 7^4). Logo, o total de fatores 7 em
10200 ! e A + B + C + D = 1698.

Como de 1 ate 10200 existem 1 numero par ( divisivel por 2 ) a cada
dois numeros segue que ha mais que 10200 / 2 = 5100 fatores 2 e, alem
disso, 5100 > 3*1698 = 5094. Igualmente, como de 1 ate 10200 existem 1
numero divisivel por 3 a cada tres numeros segue que ha mais que 10200
/ 3 = 3400 fatores 3 e, alem disso, 3400 > 2*1698 = 3396

Segue que N = 1698 e o numero procurado.

Esta e uma solucao PARA ATROPELAR A QUESTAO, isto e, resolucao
truculenta tipo forca bruta. Nao ha inteligencia aqui. Eu precisaria
ficar receptivo para receber ideias bonitas mas estou sem tempo.

Um Abracao
Paulo Santa Rita
3,0A20,120607

Em tempo : por favor, verifique se nao cometi algum erro de calculo. O
raciocinio e correto, eu garanto

Em 11/06/07, Artur Costa Steiner<[EMAIL PROTECTED]> escreveu:



Estou tentando achar uma solucoa para o seguinte, mas ainda nao consegui:

Encontrar o mair valor do ineiro n>=0 tal que (10200!)/(504^n) seja inteiro.
Nos temos que 504 = 2^3  * 3^2 *  7, assim, o quociente sera inteiro
enquanto 10200! contiver os primos 2, 3  e 7 com expoentes no maximo de 3n ,
2n e n, respectivamente. Mas nao sei se hah uma forma facil de fazer isso.

Obrigado
Artur

=
Instruções para entrar na lista, sair da lista e usar a lista em
http://www.mat.puc-rio.br/~nicolau/olimp/obm-l.html
=


RE: [obm-l] Teoria dos numeros

2007-06-11 Por tôpico Rhilbert Rivera

Olá Artur
Analise o que fiz.
Primeiro, pensando na decomposição de um fatorial em fatores primos, escrevi 
10200 nas bases 2, 3 e 7 respectivamente:
 
10200 = (1001011)2
10200 = (1111)3
10200 = (41511)7
Determinano as potências de 2, 3 e 7 na decomposição de  10200! em fatores 
primos:
 
10200-(1+1+1+1+1+1+1+1)/(2-1) = 10192 (potência do 2);
10200-(1+1+1+2+2+2+2+1)/(3-1) = 5094 ( potência do 3);
10200-(4+1+5+1+1)/(7-1) = 1698  ( potência do 7)
 
Fazendo a decomposição dessas potências em fatores primos:
 
10192 = 24 . 72 . 13  ( 10192 = 36 . 283 + 4)

5094 = 2 . 32. 283
1698 = 2. 3. 283  
 
Acredito que a melhor escolha pra n seja 283, salvo  algum engano ou algo 
melhor do que eu fiz. 
 
[[ ]]'s

 


Subject: [obm-l] Teoria dos numerosDate: Mon, 11 Jun 2007 10:27:55 -0300From: 
[EMAIL PROTECTED]: obm-l@mat.puc-rio.br



Estou tentando achar uma solucoa para o seguinte, mas ainda nao consegui:
 
Encontrar o mair valor do ineiro n>=0 tal que (10200!)/(504^n) seja inteiro. 
Nos temos que 504 = 2^3  * 3^2 *  7, assim, o quociente sera inteiro enquanto 
10200! contiver os primos 2, 3  e 7 com expoentes no maximo de 3n , 2n e n, 
respectivamente. Mas nao sei se hah uma forma facil de fazer isso.
 
Obrigado
Artur
_
Conheça o Windows Live Spaces, a rede de relacionamentos conectada ao Messenger!
http://spaces.live.com/signup.aspx

Re:[obm-l] Teoria dos numeros?

2006-08-02 Por tôpico claudio\.buffara
 




De:
[EMAIL PROTECTED]




Para:
obm-l@mat.puc-rio.br




Cópia:





Data:
Tue, 01 Aug 2006 14:37:56 -0400




Assunto:
[obm-l] Teoria dos numeros?
> Liste todos os pares (m,n) para os quais 2^m + 3^n e um quadrado perfeito.
> 
Estou supondo que m e n são inteiros não-negativos.
 
Por inspeção obtemos as soluções:
m = 0, n = 1 ==> 2^0 + 3^1 = 4
m = 3, n = 0 ==> 2^3 + 3^0 = 9
Aliás, estas são as únicas soluções com m = 0 ou n = 0.
 
Quem conhece o triângulo pitagórico (3,4,5) também acha rápido:
m = 4, n = 2 ==> 2^4 + 3^2 = 25
 
Alguns casos podem ser eliminados via congruências.
 
Por exemplo, se m >= 1 e n é ímpar, então 2^m + 3^n é ímpar.
Além disso, n ímpar ==> 3^n == 3 (mod 8). 
Logo:
m = 1 ==> 2^m + 3^n == 2 + 3 == 5 (mod 8).
m = 2 ==> 2^m + 3^n == 4 + 3 == 7 (mod 8)
m >= 3 ==> 2^m + 3^n == 0 + 3 == 3 (mod 8) 
No entanto, o quadrado de um ímpar é sempre == 1 (mod 8).
Conclusão: a única solução com n ímpar é m = 0, n = 1.
 
***
 
n é par (n = 2p, p >= 0) ==>
2^m + 3^(2p) = a^2 ==>
2^m = (a - 3^p)(a + 3^p) ==>
a - 3^p = 2^k   e   a + 3^p = 2^(m-k), com m > 2k ==>
2*3^p = 2^(m-k) - 2^k = 2^k*(2^(m-2k) - 1) ==>
3^p = 2^(k-1)*(2^(m-2k) - 1) ==>
k = 1 (fatoração única em Z) ==>
3^p = 2^(m-2) - 1 ==>
m >= 3
 
m = 3 ==> 3^p = 1 ==> p = 0 ==> n = 0 ==> (m,n) = (3,0)
m = 4 ==> 3^p = 3 ==> p = 1 ==> n = 2 ==> (m,n) = (4,2)
m >= 5 ==>
(fazendo q = m-2, de modo que q >= 3)
2^q = 3^p + 1 ==>
3^p == -1 == 7 (mod 8) ==>
não há soluções neste caso, pois 3^p == 1 ou 3 (mod 8), conforme p seja par ou ímpar
 
Logo, as únicas soluções são (0,1), (3,0) e (4,2).
 
[]s,
Claudio.


Re: [obm-l] Teoria dos numeros?

2006-08-01 Por tôpico Manuel Garcia
Boa noite,Acho que há alguns problemas com a resposta parcial abaixoOn 8/1/06, Marcelo Salhab Brogliato <[EMAIL PROTECTED]
> wrote:Olá,uma parte da resposta seria:(2a, 0)(0, 2a)onde "a" pertence aos inteiros positivos
Nenhum desses pares é solução, repare que 2^0=3^0=1.O problema proposto pode ser resolvido com uma tediosa análise de congruências nada emocionante, salvo algum engano as únicas soluções são 
m=3, n=0 (nesse caso q=3)m=4, n=2 (q=5)m=0, n=1 (q=2)Manuel Garcia
(4, 2) tb é...to tentando achar algum padrao pra isso... pq algebricamente eu nao conseguiresolver...espero ter ajudado em algoabraços,Salhab- Original Message -From: "Qwert Smith" <
[EMAIL PROTECTED]>To: Sent: Tuesday, August 01, 2006 3:37 PMSubject: [obm-l] Teoria dos numeros?
> Liste todos os pares (m,n) para os quais 2^m + 3^n e um quadrado perfeito.>>> => Instruções para entrar na lista, sair da lista e usar a lista em
> http://www.mat.puc-rio.br/~nicolau/olimp/obm-l.html> =>>
> --> No virus found in this incoming message.> Checked by AVG Free Edition.> Version: 7.1.394 / Virus Database: 268.10.5/405 - Release Date: 1/8/2006>>=
Instruções para entrar na lista, sair da lista e usar a lista emhttp://www.mat.puc-rio.br/~nicolau/olimp/obm-l.html=



Re: [obm-l] Teoria dos numeros?

2006-08-01 Por tôpico Marcelo Salhab Brogliato

Olá,
uma parte da resposta seria:

(2a, 0)
(0, 2a)
onde "a" pertence aos inteiros positivos

(4, 2) tb é...
to tentando achar algum padrao pra isso... pq algebricamente eu nao consegui 
resolver...


espero ter ajudado em algo
abraços,
Salhab



- Original Message - 
From: "Qwert Smith" <[EMAIL PROTECTED]>

To: 
Sent: Tuesday, August 01, 2006 3:37 PM
Subject: [obm-l] Teoria dos numeros?



Liste todos os pares (m,n) para os quais 2^m + 3^n e um quadrado perfeito.


=
Instruções para entrar na lista, sair da lista e usar a lista em
http://www.mat.puc-rio.br/~nicolau/olimp/obm-l.html
=


--
No virus found in this incoming message.
Checked by AVG Free Edition.
Version: 7.1.394 / Virus Database: 268.10.5/405 - Release Date: 1/8/2006




=
Instruções para entrar na lista, sair da lista e usar a lista em
http://www.mat.puc-rio.br/~nicolau/olimp/obm-l.html
=


Re: [obm-l] Teoria dos Numeros II

2006-01-29 Por tôpico Danilo Nascimento
Pense assim.  Se n+3 e n^2+3 sao cubos perfeitos, entao o seu produto tb eh.  seja m inteiro tal que m^3=(n+3)(n^2+3) desenvolvendo m^3=(n+1)^3+8  dessa forma os pares de cubos que diferem de 8 sao (-8,0) e (0,8)  entao (n+1)^3= - 8 ► n = - 3 e (n+1)^3=0 ► n= -1 para ambos n^2+3 e n+3 nao sao cubos perfeitos ◙ Klaus Ferraz <[EMAIL PROTECTED]> escreveu:1) Seja n um inteiro qualquer. Pode n+3 e n^2+3 serem cubos perfeitos?     2) Para quais inteiros n, 18(n^2+3) é cubo perfeito?  Yahoo! doce lar. Faça do Yahoo! sua homepage.  
		 
Yahoo! doce lar. Faça do Yahoo! sua homepage.

Re:[obm-l] Teoria dos Numeros II

2006-01-28 Por tôpico Danilo Nascimento
ou  veja que 18(n^2+3)=(n+3)^3-(n-3)^3  logo pelo ultimo teorema de fermat, x^n=y^n+z^n, em particular para n=3 a equacao nao possui solucao. dessa forma n+3=0 ou n-3=0 logo n= -+3."Luiz H. Barbosa" <[EMAIL PROTECTED]> escreveu:  2) Para quais inteiros n, 18(n^2+3) é cubo perfeito?      =  Vou resolver esse sem nenhuma ideia esperta:  Se 18(n^2+3) é cubo perfeito , então:     18(n^2+3) = x^3 e x>0  3.3.2(n^2+3) = x.x^2  Como x é inteiro , temos varios casos:  x=2,x=3,x=6, x=9 e x=18 e depois x=(n^2+3),x=2(n^2+3),x=3(n^2+3),x=6(n^2+3) ,x=9(n^2+3) e x=18(n^2+3) .  Se analizar ai , vai ver que o unico x possivel é x=6 , assim!
: 
 3(n^2+3) = 36 --> n = +/-3.     []'s  Luiz H. Barbosa      
		 
Yahoo! doce lar. Faça do Yahoo! sua homepage.

Re:[obm-l] Teoria dos Numeros II

2006-01-28 Por tôpico Luiz H\. Barbosa

2) Para quais inteiros n, 18(n^2+3) é cubo perfeito? 
 
=
Vou resolver esse sem nenhuma ideia esperta:
Se 18(n^2+3) é cubo perfeito , então:
 
18(n^2+3) = x^3 e x>0
3.3.2(n^2+3) = x.x^2
Como x é inteiro , temos varios casos:
x=2,x=3,x=6, x=9 e x=18 e depois x=(n^2+3),x=2(n^2+3),x=3(n^2+3),x=6(n^2+3) ,x=9(n^2+3) e x=18(n^2+3) .
Se analizar ai , vai ver que o unico x possivel é x=6 , assim:
3(n^2+3) = 36 --> n = +/-3.
 
[]'s
Luiz H. Barbosa 
 


Re: [obm-l] Teoria dos Numeros[off - topic]

2006-01-28 Por tôpico Henrique Rennó
Olá Danilo!!!

Agradeço a resposta. Acho que tem umas correções no desenvolvimento da
expressão a serem feitas.

Klaus,

Os polígonos são de 4, 6 e 10 lados e não 3, 4 e 6.

> (x+1)^3-x^3=y^2 --> desenvolva o cubo perfeito.
> 3x^2+6x+1=y^2 ---> multiplique tudo por 4
3x^2 + 3x + 1 = y^2 --> não 6x
> 12x^2+24x+4 = 4y^2---> faça o 4=3+1
12x^2 + 12x + 4 = 4y^2 --> não 24x
> 12x^2+24x+3=4y^2-1
mesmo do anterior
> 3(4x^2+8x+1)=(2y-1)(2y+1)
3(4x^2 + 4x + 1) = (2y-1)(2y+1) --> não 8x
> 2(2x+1)^2=(2y-1)(2y+1)
3(2x+1)^2 = (2y-1)(2y+1) --> não 2(2x+1)^2

Mas não entendi a seguinte parte:

> > O interessante é que para 3x^2+3x +1 =y^2 tem para solução
> > geral :
> >
> > x1! = 4y+7x+3 e y1 = 7y+12x+6 com x e y conhecidos . Exemplo :
> > x1 = 104 e y1 =181

Abraços,

--
Henrique

=
Instruções para entrar na lista, sair da lista e usar a lista em
http://www.mat.puc-rio.br/~nicolau/olimp/obm-l.html
=


Re: [obm-l] Teoria dos Numeros[off - topic]

2006-01-28 Por tôpico Danilo Nascimento
Ola Henrique,     (x+1)^3-x^3=y^2 --> desenvolva o cubo perfeito.  3x^2+6x+1=y^2 ---> multiplique tudo por 4  12x^2+24x+4 = 4y^2---> faça o 4=3+1  12x^2+24x+3=4y^2-1  3(4x^2+8x+1)=(2y-1)(2y+1)  2(2x+1)^2=(2y-1)(2y+1)  Dai use que (2y-1)(2y+1) sao primos entre si.  Veja q letra b) nao pode ocorrer porque ficaria 3c^2+2=d^2  dai eh so vc olhar a expressao no mod 3. como todo quadrado eh congruente a 0 ou 1 mod 3. logo nao pode ser.   Henrique Rennó <[EMAIL PROTECTED]> escreveu:  Olá Klaus e Carlos VictorFiz duas observações nesta questão e gostaria que vocês me ajudassem.Klaus, eu tinha lhe enviado por e-mail um arquivo .doc do word com umapossível solução para um exercíci!
o que
 fosse havia postado sobre acharum ângulo de um triângulo formado pelos lados dos polígonos regularesde 3,4 e 6 lados inscritos num círculo. Você recebeu??? Se não, meavise que te envio novamente.Abraços!!!> (x+1)^3 - x^3 = y^2 , onde 3(2x+1)^2 = (2y-1)(2y+1) . Observe queNão entendi essa expressão: 3(2x+1)^2 = (2y-1)(2y+1). O que foipensado para formar ela???> podemos concluir que :>> a) Ou 2y-1 = a^2 e 2y+1 = 3b^2>> b) Ou 2y-1 = 3c^2 e 2y+1 = d^2 .>>> Observe que 3b^2 = a^2 +2 é a única que pode ocorrer e, como> a é ímpar , podemos escrever>> a = 2t +1 e 4y = 2(a^2+1) implicando y = t^2 + (t+1)^2 , ok ?>> OBS : (1) Esta questão se encontra no Livro POWER PLAY de> EDWARD J. BARBEAU da MAA ; inclusive com a solução acima>> (2) O interessante é que para 3x^2+3x +1 =y^2 tem para solução> geral :>> x1!
 =
 4y+7x+3 e y1 = 7y+12x+6 com x e y conhecidos . Exemplo :> x1 = 104 e y1 =181 ; Lindo não é ?>>> []´s Carlos Victor>> At 20:23 24/1/2006, Klaus Ferraz wrote:>Esse enunciado não deveria ser: Mostre que "se" a diferençaPorque, por exemplo, 5^3 - 4^3 = 125 - 64 = 61. Não existe raizquadrada inteira de 61.> Mostre que a diferença entre os cubos de dois numeros inteiros consecutivos> é igual ao quadrado de um inteiro, entao esse inteiro é igual a soma dos> quadrados ! de dois inteiros consecutivos.> Ex: 8^3-7^3=169. 2^2+3^2=13.>> Grato.>>> Yahoo! doce lar. Faça do Yahoo! sua homepage.>>> > Yahoo! doce lar. Faça do Yahoo! sua homepage.>> >>> No virus found in this incoming message.> C!
hecked by
 AVG Free Edition.> Version: 7.1.375 / Virus Database: 267.14.22/238 - Release Date: 23/1/2006>--Henrique=Instruções para entrar na lista, sair da lista e usar a lista emhttp://www.mat.puc-rio.br/~nicolau/olimp/obm-l.html=  __Faça ligações para outros computadores com o novo Yahoo! Messenger http://br.beta.messenger.yahoo.com/ 

Re: [obm-l] Teoria dos Numeros[off - topic]

2006-01-27 Por tôpico Henrique Rennó
Olá Klaus e Carlos Victor

Fiz duas observações nesta questão e gostaria que vocês me ajudassem.

Klaus, eu tinha lhe enviado por e-mail um arquivo .doc do word com uma
possível solução para um exercício que fosse havia postado sobre achar
um ângulo de um triângulo formado pelos lados dos polígonos regulares
de 3,4 e 6 lados inscritos num círculo. Você recebeu??? Se não, me
avise que te envio novamente.

Abraços!!!

> (x+1)^3 - x^3 = y^2   , onde  3(2x+1)^2 = (2y-1)(2y+1) . Observe  que

Não entendi essa expressão: 3(2x+1)^2 = (2y-1)(2y+1). O que foi
pensado para formar ela???

> podemos  concluir  que  :
>
> a) Ou  2y-1 = a^2e  2y+1 = 3b^2
>
> b) Ou  2y-1 = 3c^2e  2y+1 = d^2.
>
>
> Observe  que 3b^2 =  a^2  +2   é  a única  que  pode  ocorrer  e,  como
>   a é  ímpar   , podemos  escrever
>
> a = 2t +1   e   4y = 2(a^2+1)   implicandoy =   t^2  + (t+1)^2  , ok  ?
>
>  OBS : (1) Esta  questão  se  encontra  no  Livro   POWER  PLAYde
> EDWARD J.  BARBEAU  da  MAA ; inclusive  com  a  solução  acima
>
> (2)  O  interessante  é  que  para  3x^2+3x +1 =y^2   tem  para  solução
> geral   :
>
> x1 = 4y+7x+3   e  y1  =  7y+12x+6  com   x  e  y conhecidos  . Exemplo  :
> x1 = 104  e y1  =181 ; Lindo  não  é  ?
>
>
> []´s   Carlos  Victor
>
>
>
>
>
> At 20:23 24/1/2006, Klaus Ferraz wrote:
>

Esse enunciado não deveria ser: Mostre que "se" a diferença
Porque, por exemplo, 5^3 - 4^3 = 125 - 64 = 61. Não existe raiz
quadrada inteira de 61.

> Mostre que a diferença entre os cubos de dois numeros inteiros consecutivos
> é igual ao quadrado de um inteiro, entao esse inteiro é igual a soma dos
> quadrados ! de dois inteiros consecutivos.
> Ex: 8^3-7^3=169.   2^2+3^2=13.
>
> Grato.
>
>
> Yahoo! doce lar. Faça do Yahoo! sua homepage.
>
>
>  
>  Yahoo! doce lar. Faça do Yahoo! sua homepage.
>
>  
>
>
> No virus found in this incoming message.
> Checked by AVG Free Edition.
> Version: 7.1.375 / Virus Database: 267.14.22/238 - Release Date: 23/1/2006
>
>
>
>
>


--
Henrique

=
Instruções para entrar na lista, sair da lista e usar a lista em
http://www.mat.puc-rio.br/~nicolau/olimp/obm-l.html
=


Re: [obm-l] Teoria dos Numeros[off - topic]

2006-01-26 Por tôpico Igor Castro



na www.amazon.com
 
 

  - Original Message - 
  From: 
  Klaus 
  Ferraz 
  To: obm-l@mat.puc-rio.br 
  Sent: Thursday, January 26, 2006 7:00 
  PM
  Subject: Re: [obm-l] Teoria dos 
  Numeros[off - topic]
  
  Vlw. Onde consigo esse livro, POWER  
  PLAY    de   EDWARD J.  BARBEAU  da  
  MAA Carlos Victor <[EMAIL PROTECTED]> 
  escreveu: 
  Olá  
Klauss ,(x+1)^3 - x^3 = y^2   , onde  3(2x+1)^2 = 
(2y-1)(2y+1) . Observe  que  podemos  concluir  
que  :a) Ou  2y-1 = a^2    e  2y+1 = 
3b^2  b) Ou  2y-1 = 3c^2    e  2y+1 = 
d^2    .Observe  que 
3b^2 =  a^2  +2   é  a única  que  
pode  ocorrer  e,  como    a é  
ímpar   , podemos  escrever  a = 2t 
+1   e   4y = 2(a^2+1)   
implicando    y =   t^2  + (t+1)^2  , 
ok  ? OBS : (1) Esta  questão  se  
encontra  no  Livro   POWER  PLAY    
de   EDWARD J.  BARBEAU  da  MAA ; inclusive  
com  a  solução  acima (2)  O  
interessante  é  que  para  3x^2+3x +1 =y^2   
tem  para  solução  geral   :x1 = 
4y+7x+3   e  y1  =  7y+12x+6  com   
x  e  y conhecidos  . Exemplo  :  x1 = 104  e 
y1  =181 ; Lindo  não  é  ?[]´s   
Carlos  VictorAt 20:23 24/1/2006, Klaus Ferraz 
wrote:
Mostre que a diferença entre os 
  cubos de dois numeros inteiros consecutivos é igual ao quadrado de um 
  inteiro, entao esse inteiro é igual a soma dos quadrados ! de dois 
  inteiros consecutivos.Ex: 8^3-7^3=169.   
  2^2+3^2=13. Grato.Yahoo! doce lar. Faça 
  do Yahoo! sua homepage. 
  
  
  Yahoo! doce lar. Faça 
  do Yahoo! sua homepage.
  
  

  No virus found in this incoming message.Checked by AVG Free 
  Edition.Version: 7.1.375 / Virus Database: 267.14.22/238 - Release Date: 
  23/1/2006


Re: [obm-l] Teoria dos Numeros[off - topic]

2006-01-26 Por tôpico Klaus Ferraz
Vlw. Onde consigo esse livro, POWER  PLAY    de   EDWARD J.  BARBEAU  da  MAA Carlos Victor <[EMAIL PROTECTED]> escreveu:  Olá  Klauss ,(x+1)^3 - x^3 = y^2   , onde  3(2x+1)^2 = (2y-1)(2y+1) . Observe  que  podemos  concluir  que  :a) Ou  2y-1 = a^2    e  2y+1 = 3b^2  b) Ou  2y-1 = 3c^2    e  2y+1 = d^2    .Observe  que 3b^2 =  a^2  +2   é  a única  que  pode  ocorrer  e,  como    a é  ímpar   , podemos  escrever  a = 2t +1   e   4y = 2(a^2+1)   implicando    y
 =   t^2  + (t+1)^2  , ok  ? OBS : (1) Esta  questão  se  encontra  no  Livro   POWER  PLAY    de   EDWARD J.  BARBEAU  da  MAA ; inclusive  com  a  solução  acima (2)  O  interessante  é  que  para  3x^2+3x +1 =y^2   tem  para  solução  geral   :x1 = 4y+7x+3   e  y1  =  7y+12x+6  com   x  e  y conhecidos  . Exemplo  :  x1 = 104  e y1  =181 ; Lindo  não  é  ?[]´s   Carlos  VictorAt 20:23 24/1/2006, Klaus Ferraz wrote:  Mostre que a diferença entre os cubos de dois numeros inteiros consecutivos é igual ao quadrado de um inteiro, entao esse inteiro é igual a soma dos quadrados !
de dois
 inteiros consecutivos.Ex: 8^3-7^3=169.   2^2+3^2=13. Grato.Yahoo! doce lar. Faça do Yahoo! sua homepage. 
		 
Yahoo! doce lar. Faça do Yahoo! sua homepage.

Re: [obm-l] Teoria dos Numeros

2006-01-26 Por tôpico Carlos Victor


Olá  Klauss ,
(x+1)^3 - x^3 = y^2   , onde  3(2x+1)^2 = (2y-1)(2y+1) .
Observe  que  podemos  concluir  que  
:
a) Ou  2y-1 = a^2    e  2y+1 = 3b^2 

b) Ou  2y-1 = 3c^2    e  2y+1 =
d^2    .

Observe  que 3b^2 =  a^2 
+2   é  a única  que  pode  ocorrer 
e,  como    a é  ímpar   ,
podemos  escrever  
a = 2t +1   e   4y = 2(a^2+1)  
implicando    y =   t^2  + (t+1)^2  ,
ok  ?
 OBS : (1) Esta  questão  se  encontra  no 
Livro   POWER  PLAY    de  
EDWARD J.  BARBEAU  da  MAA ; inclusive  com 
a  solução  acima 
(2)  O  interessante  é  que  para  3x^2+3x
+1 =y^2   tem  para  solução  geral  
:
x1 = 4y+7x+3   e  y1  =  7y+12x+6 
com   x  e  y conhecidos  . Exemplo 
:  x1 = 104  e y1  =181 ; Lindo  não  é 
?

[]´s   Carlos  Victor


At 20:23 24/1/2006, Klaus Ferraz wrote:
Mostre que a diferença entre os
cubos de dois numeros inteiros consecutivos é igual ao quadrado de um
inteiro, entao esse inteiro é igual a soma dos quadrados de dois inteiros
consecutivos.
Ex: 8^3-7^3=169.   2^2+3^2=13.
 
Grato.

Yahoo! doce lar.
Faça
do Yahoo! sua homepage. 



Re:[obm-l] teoria dos numeros

2005-10-19 Por tôpico claudio\.buffara
Que tal 7 e 19?
7 = 4*1 + 3  e  19 = 4*4 + 3
mdc(1+1,4+1) = 1.
 
[]s,
Claudio.
 




De:
[EMAIL PROTECTED]




Para:
obm-l@mat.puc-rio.br




Cópia:





Data:
Wed, 19 Oct 2005 16:01:03 -0200




Assunto:
[obm-l] teoria dos numeros
> boa tarde a todos, quem me ajuda com esse?
> 
> sabemos que existem infinitos numeros primos da forma 4k + 3. dado um 
> inteiro b e sendo S o conjunto de todos os primos da forma p = 4k + 3 , onde 
> p não divide b. a questão é: existem dois primos (4k + 3) e (4q + 3) em S de 
> tal forma que (k + 1) e (q + 1) são primos entre si?
> 
> abraços
> 
> _
> Chegou o que faltava: MSN Acesso Grátis. Instale Já! 
> http://www.msn.com.br/discador
> 
> =
> Instruções para entrar na lista, sair da lista e usar a lista em
> http://www.mat.puc-rio.br/~nicolau/olimp/obm-l.html
> =
> 


[obm-l] Re: [obm-l] Teoria dos Numeros-Soluçao de um Hojoo Lee

2004-05-03 Por tôpico Domingos Jr.
> Considere alguns primos da forma 2pk+1, no conjunto { p_1 ,..., p_n } .

vc deve mostrar que existe um conjunto inicial não vazio para poder
construir um conjunto infinito a partir dele.

[ ]'s

=
Instruções para entrar na lista, sair da lista e usar a lista em
http://www.mat.puc-rio.br/~nicolau/olimp/obm-l.html
=


[obm-l] Re: [obm-l] Teoria dos Numeros-Soluçao de um Hojoo Lee

2004-05-03 Por tôpico Domingos Jr.
"Seja p um primo impar.
Prove que existem infinitos primos x tais que 2p divide x-1".

considere a PA {(2p)n + 1 : n pertence a Z}
como mdc(2p, 1) = 1 temos, pelo seu teorema (Dirichlet) que tal PA possui
infinitos primos.
ou seja, este problema é um caso particular do super-canhão-teorema de PAs.

[ ]'s

=
Instruções para entrar na lista, sair da lista e usar a lista em
http://www.mat.puc-rio.br/~nicolau/olimp/obm-l.html
=


Re: [obm-l] Teoria dos Numeros

2004-04-20 Por tôpico Johann Peter Gustav Lejeune Dirichlet
Qualquer coincidencia e mera semelhança...Claudio Buffara <[EMAIL PROTECTED]> wrote:
Analise os 7 casos possiveis, a == 0, 1, 2, 3, 4, 5 e 6 (mod 7). Alias, nemprecisa analisar todos, jah que k^2 == (7-k)^2 (mod 7). A solucao saifacilmente.Sobre a generalizacao, suponhamos que a condicao seja:a, b pertencem a X ==> ab + k pertence a X, com k = inteiro fixo.Entao: a pertence a X ==>a^(n+2) + k*(a^n + a^(n-1)+...+ a + 1) =a^(n+2) + k*(a^(n+1) - 1)/(a - 1) pertence a X para todo n.Definimos a sequencia (b(n)) da seguinte forma:b(0) = ab(n) = a*b(n-1) + k, para n >= 1.Eh claro que b(1) = a^2 + k = p > a e tambem que mdc(a,p) = 1 (jah que a e psao primos distintos).Tambem eh claro que b(n) = a^(n+1) + k*(a^n - 1)/(a - 1) pertence a X.Em particular, b(p-1) = a^p + k*(a^(p-1) - 1)/(a - 1).Mas, pelo pequeno teorema de Fermat, a^p == a e a^(p-1) == 1 (mod
 p).Logo, b(p-1) == a = b(0) (mod p) ==>b(p) = a*b(p-1) + k == a*b(0) + k = b(1) = p (mod p) ==>p = b(1) divide b(p) ==>b(p) eh composto ==>contradicao ==>X eh vazio.Qualquer semelhanca coma demonstracao do Gugu NAO eh mera coincidencia.[]s,Claudio.on 20.04.04 13:46, Johann Peter Gustav Lejeune Dirichlet at[EMAIL PROTECTED] wrote:> Onde esta ela?> Alias sera que da para generalizar esse quatro?> > --- Claudio Buffara> <[EMAIL PROTECTED]>escreveu: > on> 17.04.04 10:56, Johann Peter Gustav Lejeunne>> Dirichlet at>> [EMAIL PROTECTED] wrote:>> >> Seja X o conjunto dos primos tais que se a e b>> sao dois elementos dele entao>> ab+4 e a^2+4 tambem estao.Prove ou disprove: X>> e vazio>> >> Alem da solucao do Gugu, existe uma
 outra,>> encontrada pelo Carlos Yuzo Shine>> usando congruencia mod 7.>> >> >> []s,>> Claudio.>> >> =Instruções para entrar na lista, sair da lista e usar a lista emhttp://www.mat.puc-rio.br/~nicolau/olimp/obm-l.html=

TRANSIRE SVVM PECTVS MVNDOQVE POTIRI
CONGREGATI EX TOTO ORBE MATHEMATICI OB SCRIPTA INSIGNIA TRIBVERE
Fields Medal(John Charles Fields)Yahoo! Messenger - Fale com seus amigos online. Instale agora!

Re: [obm-l] Teoria dos Numeros

2004-04-20 Por tôpico Claudio Buffara
Analise os 7 casos possiveis, a == 0, 1, 2, 3, 4, 5 e 6 (mod 7). Alias, nem
precisa analisar todos, jah que k^2 == (7-k)^2 (mod 7). A solucao sai
facilmente.



Sobre a generalizacao, suponhamos que a condicao seja:
a, b pertencem a X ==> ab + k pertence a X, com k = inteiro fixo.

Entao: 
a pertence a X ==>
a^(n+2) + k*(a^n + a^(n-1)+...+ a + 1) =
a^(n+2) + k*(a^(n+1) - 1)/(a - 1) pertence a X para todo n.

Definimos a sequencia (b(n)) da seguinte forma:
b(0) = a
b(n) = a*b(n-1) + k, para n >= 1.

Eh claro que b(1) = a^2 + k = p > a e tambem que mdc(a,p) = 1 (jah que a e p
sao primos distintos).
Tambem eh claro que b(n) = a^(n+1) + k*(a^n - 1)/(a - 1) pertence a X.

Em particular, b(p-1) = a^p + k*(a^(p-1) - 1)/(a - 1).

Mas, pelo pequeno teorema de Fermat, a^p == a  e  a^(p-1) == 1 (mod p).

Logo, b(p-1) == a = b(0) (mod p) ==>
b(p) = a*b(p-1) + k == a*b(0) + k = b(1) = p (mod p) ==>

p = b(1) divide b(p) ==>
b(p) eh composto ==>
contradicao ==>
X eh vazio.

Qualquer semelhanca coma demonstracao do Gugu NAO eh mera coincidencia.

[]s,
Claudio.

on 20.04.04 13:46, Johann Peter Gustav Lejeune Dirichlet at
[EMAIL PROTECTED] wrote:

> Onde esta ela?
> Alias sera que da para generalizar esse quatro?
> 
> --- Claudio Buffara
> <[EMAIL PROTECTED]> escreveu: > on
> 17.04.04 10:56, Johann Peter Gustav Lejeune
>> Dirichlet at
>> [EMAIL PROTECTED] wrote:
>> 
>> Seja X o conjunto dos primos tais que se a e b
>> sao dois elementos dele entao
>> ab+4 e a^2+4 tambem estao.Prove ou disprove: X
>> e vazio
>> 
>> Alem da solucao do Gugu, existe uma outra,
>> encontrada pelo Carlos Yuzo Shine
>> usando congruencia mod 7.
>> 
>> 
>> []s,
>> Claudio.
>> 
>> 

=
Instruções para entrar na lista, sair da lista e usar a lista em
http://www.mat.puc-rio.br/~nicolau/olimp/obm-l.html
=


Re: [obm-l] Teoria dos Numeros

2004-04-20 Por tôpico Johann Peter Gustav Lejeune Dirichlet
A soluçao do Gugu, como ja era de se esperar, foi
demais!!
 --- Johann Peter Gustav Lejeune Dirichlet
<[EMAIL PROTECTED]> escreveu: > Uma
resposta para o Claudio:
> 
> Este problema eu propus logo quando eu entrei
> na
> lista.Ninguem tinha mandado nada sobre
> isso.resolvi mandar de novo agora que vi em uns
> papeis de matematica olimpica que eu guardava.
> Eu ate um tempinho atras so tinha limitado os
> caras em algumas congruencias,mas nada
> conclusivo.
> 
> 
>  --- Claudio Buffara
> <[EMAIL PROTECTED]> escreveu: > on
> 19.04.04 12:54, Ricardo Bittencourt at
> > [EMAIL PROTECTED] wrote:
> > 
> > > Johann Peter Gustav Lejeune Dirichlet
> wrote:
> > > 
> > > (a,b => ab+4 e a^2+4)
> > >> Mas espere, 6m-1=m-1 mod 5.Logo
> > (6m-1)(6n-1)+4=mn-m-n mod 5.Sera que da
> > >> para arrancar alguem mod 5?
> > >> Se mn=m+n mod 5 entao nao da primo
> > > 
> > > Eu já consegui mostrar que todos os
> elementos
> > do
> > > conjunto são da forma 30k+23, mas ainda não
> > cheguei em
> > > nenhuma contradição.
> > > 
> > Mais ainda: eles tem que ser da forma 60k +
> 53.
> > Infelizmente, eu acho que
> > esse caminho nao vai levar aa solucao, mas
> pelo
> > menos aumenta a minha
> > conviccao de que X eh vazio.
> > 
> > Alias, uma pergunta pro Dirichlet: de onde
> voce
> > tirou esse problema?
> > 
> > []s,
> > Claudio.
> > 
> > 
> >
>
=
> > Instruções para entrar na lista, sair da
> lista
> > e usar a lista em
> >
>
http://www.mat.puc-rio.br/~nicolau/olimp/obm-l.html
> >
>
=r/~nicolau/olimp/obm-l.html
> >
>
=
> 
> 
> =
> 
> TRANSIRE SVVM PECTVS MVNDOQVE POTIRI
> 
> CONGREGATI EX TOTO ORBE MATHEMATICI OB SCRIPTA
> INSIGNIA TRIBVERE
> 
> Fields Medal(John Charles Fields)
> 
> 
> 
> 
>
__
> 
> Yahoo! Messenger - Fale com seus amigos online.
> Instale agora! 
> http://br.download.yahoo.com/messenger/
>
=
> Instruções para entrar na lista, sair da lista
> e usar a lista em
>
http://www.mat.puc-rio.br/~nicolau/olimp/obm-l.html
>
= 

=

TRANSIRE SVVM PECTVS MVNDOQVE POTIRI

CONGREGATI EX TOTO ORBE MATHEMATICI OB SCRIPTA INSIGNIA TRIBVERE

Fields Medal(John Charles Fields)




__

Yahoo! Messenger - Fale com seus amigos online. Instale agora! 
http://br.download.yahoo.com/messenger/
=
Instruções para entrar na lista, sair da lista e usar a lista em
http://www.mat.puc-rio.br/~nicolau/olimp/obm-l.html
=


Re: [obm-l] Teoria dos Numeros

2004-04-20 Por tôpico Johann Peter Gustav Lejeune Dirichlet
Onde esta ela?
Alias sera que da para generalizar esse quatro?

 --- Claudio Buffara
<[EMAIL PROTECTED]> escreveu: > on
17.04.04 10:56, Johann Peter Gustav Lejeune
> Dirichlet at
> [EMAIL PROTECTED] wrote:
> 
> Seja X o conjunto dos primos tais que se a e b
> sao dois elementos dele entao
> ab+4 e a^2+4 tambem estao.Prove ou disprove: X
> e vazio
> 
> Alem da solucao do Gugu, existe uma outra,
> encontrada pelo Carlos Yuzo Shine
> usando congruencia mod 7.
> 
> 
> []s,
> Claudio.
> 
>  

=

TRANSIRE SVVM PECTVS MVNDOQVE POTIRI

CONGREGATI EX TOTO ORBE MATHEMATICI OB SCRIPTA INSIGNIA TRIBVERE

Fields Medal(John Charles Fields)




__

Yahoo! Messenger - Fale com seus amigos online. Instale agora! 
http://br.download.yahoo.com/messenger/
=
Instruções para entrar na lista, sair da lista e usar a lista em
http://www.mat.puc-rio.br/~nicolau/olimp/obm-l.html
=


Re: [obm-l] Teoria dos Numeros

2004-04-20 Por tôpico Johann Peter Gustav Lejeune Dirichlet
Uma resposta para o Claudio:

Este problema eu propus logo quando eu entrei na
lista.Ninguem tinha mandado nada sobre
isso.resolvi mandar de novo agora que vi em uns
papeis de matematica olimpica que eu guardava.
Eu ate um tempinho atras so tinha limitado os
caras em algumas congruencias,mas nada
conclusivo.


 --- Claudio Buffara
<[EMAIL PROTECTED]> escreveu: > on
19.04.04 12:54, Ricardo Bittencourt at
> [EMAIL PROTECTED] wrote:
> 
> > Johann Peter Gustav Lejeune Dirichlet wrote:
> > 
> > (a,b => ab+4 e a^2+4)
> >> Mas espere, 6m-1=m-1 mod 5.Logo
> (6m-1)(6n-1)+4=mn-m-n mod 5.Sera que da
> >> para arrancar alguem mod 5?
> >> Se mn=m+n mod 5 entao nao da primo
> > 
> > Eu já consegui mostrar que todos os elementos
> do
> > conjunto são da forma 30k+23, mas ainda não
> cheguei em
> > nenhuma contradição.
> > 
> Mais ainda: eles tem que ser da forma 60k + 53.
> Infelizmente, eu acho que
> esse caminho nao vai levar aa solucao, mas pelo
> menos aumenta a minha
> conviccao de que X eh vazio.
> 
> Alias, uma pergunta pro Dirichlet: de onde voce
> tirou esse problema?
> 
> []s,
> Claudio.
> 
> 
>
=
> Instruções para entrar na lista, sair da lista
> e usar a lista em
>
http://www.mat.puc-rio.br/~nicolau/olimp/obm-l.html
>
=r/~nicolau/olimp/obm-l.html
>
= 

=

TRANSIRE SVVM PECTVS MVNDOQVE POTIRI

CONGREGATI EX TOTO ORBE MATHEMATICI OB SCRIPTA INSIGNIA TRIBVERE

Fields Medal(John Charles Fields)




__

Yahoo! Messenger - Fale com seus amigos online. Instale agora! 
http://br.download.yahoo.com/messenger/
=
Instruções para entrar na lista, sair da lista e usar a lista em
http://www.mat.puc-rio.br/~nicolau/olimp/obm-l.html
=


Re: [obm-l] Teoria dos Numeros

2004-04-20 Por tôpico Johann Peter Gustav Lejeune Dirichlet
Pode-se dizer que sim.Eu preferi escrever desse
jeito para nao dar margem a duvidas
 --- Carlos Gustavo Tamm de Araujo Moreira
<[EMAIL PROTECTED]> escreveu: >Caros Claudio e
Dirichlet,
>Bacana esse problema. 
>Vamos la': Dadas essas condicoes, se a
> pertence a X entao
>
b(n)=a^(n+2)+4.(a^n+a^(n-1)+...+a+1)=a^(n+2)+4.(a^(n+1)-1)/(a-1)
> pertence a
> X para todo n, mas para todo primo q (digamos
> q=b(0)=a^2+4), b(n) (mod q) e'
> periodica com periodo divisor de q-1, pelo
> pequeno teorema de Fermat, donde,
> em particular, b(a^2+3) e' multiplo de
> b(0)=a^2+4, e como claramente
> b(a^2+3) > a^2+4, b(a^2+3) nao e' primo,
> absurdo. Assim, X tem que ser vazio.
>Abracos,
>  Gugu
> 
> Obs.: A primeira condicao e' um caso particular
> da segunda, nao ?
>  
> >
> >on 19.04.04 12:54, Ricardo Bittencourt at
> [EMAIL PROTECTED] wrote:
> >
> >> Johann Peter Gustav Lejeune Dirichlet wrote:
> >> 
> >> (a,b => ab+4 e a^2+4)
> >>> Mas espere, 6m-1=m-1 mod 5.Logo
> (6m-1)(6n-1)+4=mn-m-n mod 5.Sera que da
> >>> para arrancar alguem mod 5?
> >>> Se mn=m+n mod 5 entao nao da primo
> >> 
> >> Eu já consegui mostrar que todos os
> elementos do
> >> conjunto são da forma 30k+23, mas ainda não
> cheguei em
> >> nenhuma contradição.
> >> 
> >Mais ainda: eles tem que ser da forma 60k +
> 53. Infelizmente, eu acho que
> >esse caminho nao vai levar aa solucao, mas
> pelo menos aumenta a minha
> >conviccao de que X eh vazio.
> >
> >Alias, uma pergunta pro Dirichlet: de onde
> voce tirou esse problema?
> >
> >[]s,
> >Claudio.
> >
> >
>
>=
> >Instruções para entrar na lista, sair da lista
> e usar a lista em
>
>http://www.mat.puc-rio.br/~nicolau/olimp/obm-l.html
>
>=
> >
> 
>
=
> Instruções para entrar na lista, sair da lista
> e usar a lista em
>
http://www.mat.puc-rio.br/~nicolau/olimp/obm-l.html
>
= 

=

TRANSIRE SVVM PECTVS MVNDOQVE POTIRI

CONGREGATI EX TOTO ORBE MATHEMATICI OB SCRIPTA INSIGNIA TRIBVERE

Fields Medal(John Charles Fields)




__

Yahoo! Messenger - Fale com seus amigos online. Instale agora! 
http://br.download.yahoo.com/messenger/
=
Instruções para entrar na lista, sair da lista e usar a lista em
http://www.mat.puc-rio.br/~nicolau/olimp/obm-l.html
=


Re: [obm-l] Teoria dos Numeros

2004-04-20 Por tôpico Claudio Buffara
Title: Re: [obm-l] Teoria dos Numeros



on 17.04.04 10:56, Johann Peter Gustav Lejeune Dirichlet at [EMAIL PROTECTED] wrote:

Seja X o conjunto dos primos tais que se a e b sao dois elementos dele entao ab+4 e a^2+4 tambem estao.Prove ou disprove: X e vazio

Alem da solucao do Gugu, existe uma outra, encontrada pelo Carlos Yuzo Shine usando congruencia mod 7.


[]s,
Claudio.





Re: [obm-l] Teoria dos Numeros

2004-04-19 Por tôpico Carlos Gustavo Tamm de Araujo Moreira
   Caros Claudio e Dirichlet,
   Bacana esse problema. 
   Vamos la': Dadas essas condicoes, se a pertence a X entao
b(n)=a^(n+2)+4.(a^n+a^(n-1)+...+a+1)=a^(n+2)+4.(a^(n+1)-1)/(a-1) pertence a
X para todo n, mas para todo primo q (digamos q=b(0)=a^2+4), b(n) (mod q) e'
periodica com periodo divisor de q-1, pelo pequeno teorema de Fermat, donde,
em particular, b(a^2+3) e' multiplo de b(0)=a^2+4, e como claramente
b(a^2+3) > a^2+4, b(a^2+3) nao e' primo, absurdo. Assim, X tem que ser vazio.
   Abracos,
 Gugu

Obs.: A primeira condicao e' um caso particular da segunda, nao ?
 
>
>on 19.04.04 12:54, Ricardo Bittencourt at [EMAIL PROTECTED] wrote:
>
>> Johann Peter Gustav Lejeune Dirichlet wrote:
>> 
>> (a,b => ab+4 e a^2+4)
>>> Mas espere, 6m-1=m-1 mod 5.Logo (6m-1)(6n-1)+4=mn-m-n mod 5.Sera que da
>>> para arrancar alguem mod 5?
>>> Se mn=m+n mod 5 entao nao da primo
>> 
>> Eu já consegui mostrar que todos os elementos do
>> conjunto são da forma 30k+23, mas ainda não cheguei em
>> nenhuma contradição.
>> 
>Mais ainda: eles tem que ser da forma 60k + 53. Infelizmente, eu acho que
>esse caminho nao vai levar aa solucao, mas pelo menos aumenta a minha
>conviccao de que X eh vazio.
>
>Alias, uma pergunta pro Dirichlet: de onde voce tirou esse problema?
>
>[]s,
>Claudio.
>
>
>=
>Instruções para entrar na lista, sair da lista e usar a lista em
>http://www.mat.puc-rio.br/~nicolau/olimp/obm-l.html
>=
>

=
Instruções para entrar na lista, sair da lista e usar a lista em
http://www.mat.puc-rio.br/~nicolau/olimp/obm-l.html
=


Re: [obm-l] Teoria dos Numeros

2004-04-19 Por tôpico Claudio Buffara
on 19.04.04 12:54, Ricardo Bittencourt at [EMAIL PROTECTED] wrote:

> Johann Peter Gustav Lejeune Dirichlet wrote:
> 
> (a,b => ab+4 e a^2+4)
>> Mas espere, 6m-1=m-1 mod 5.Logo (6m-1)(6n-1)+4=mn-m-n mod 5.Sera que da
>> para arrancar alguem mod 5?
>> Se mn=m+n mod 5 entao nao da primo
> 
> Eu já consegui mostrar que todos os elementos do
> conjunto são da forma 30k+23, mas ainda não cheguei em
> nenhuma contradição.
> 
Mais ainda: eles tem que ser da forma 60k + 53. Infelizmente, eu acho que
esse caminho nao vai levar aa solucao, mas pelo menos aumenta a minha
conviccao de que X eh vazio.

Alias, uma pergunta pro Dirichlet: de onde voce tirou esse problema?

[]s,
Claudio.


=
Instruções para entrar na lista, sair da lista e usar a lista em
http://www.mat.puc-rio.br/~nicolau/olimp/obm-l.html
=


Re: [obm-l] Teoria dos Numeros

2004-04-19 Por tôpico Ricardo Bittencourt
Johann Peter Gustav Lejeune Dirichlet wrote:

(a,b => ab+4 e a^2+4)
Mas espere, 6m-1=m-1 mod 5.Logo (6m-1)(6n-1)+4=mn-m-n mod 5.Sera que da 
para arrancar alguem mod 5?
Se mn=m+n mod 5 entao nao da primo
Eu já consegui mostrar que todos os elementos do
conjunto são da forma 30k+23, mas ainda não cheguei em
nenhuma contradição.

Ricardo Bittencourt   http://www.mundobizarro.tk
[EMAIL PROTECTED]   "tenki ga ii kara sanpo shimashou"
-- União contra o forward - crie suas proprias piadas --
=
Instruções para entrar na lista, sair da lista e usar a lista em
http://www.mat.puc-rio.br/~nicolau/olimp/obm-l.html
=


Re: [obm-l] Teoria dos Numeros

2004-04-18 Por tôpico Johann Peter Gustav Lejeune Dirichlet
Mas espere, 6m-1=m-1 mod 5.Logo (6m-1)(6n-1)+4=mn-m-n mod 5.Sera que da para arrancar alguem mod 5?
Se mn=m+n mod 5 entao nao da primo
Claudio Buffara <[EMAIL PROTECTED]> wrote:
on 17.04.04 10:56, Johann Peter Gustav Lejeune Dirichlet at [EMAIL PROTECTED] wrote:
Seja X o conjunto dos primos tais que se a e b sao dois elementos dele entao ab+4 e a^2+4 tambem estao.Prove ou disprove: X e vazioInicio de solucao:  Suponhamos que X <> vazio e seja n um elemento de X.Eh facil ver que n > 2.Alem disso, n > 3, pois se 3 pertencesse a X, teriamos:3^2 + 4 = 13 em X ==>3*13 + 4 = 43 em X ==>13^2 + 4 = 173 em X ==>43*173 + 4 = 7443 em X ==>contradicao, pois 7443 = 3*2481 eh composto.Agora, um primo p > 3 eh da forma 3k + 1 ou 3k + 2.Se p = 3k + 1 pertence a X, entao:p^2 + 4   e   p*(p^2 + 4) + 4 = p^3 + 4p + 4 pertencem a X.Mas p^3 + 4p + 4 = (p^3 - p) + 3*(p + 1) + (2p + 1). Alem disso:p^3 - p eh multiplo de 3, pelo pequeno teorema de Fermat;3*(p + 1) eh multiplo de 3 (obviamente);2p + 1 = 2*(3k + 1) + 1 = 3*(2k + 1) eh multiplo de 3.Ou seja, p*(p^2 + 4) + 4 eh multiplo de 3 e, portanto, c!
 omposto
 ==>contradicao ==>nenhum primo da forma 3k + 1 pertence a X.Em suma, se algum primo pertence a X, ele serah da forma 3k + 2.Mais ainda: se k for par (por exemplo, k = 2m, com m > 0), teremos que:3k + 2 = 3*(2m) + 2 = 2*(3m + 1) eh par e, portanto, nao pode pertencer a X.Logo, se algum primo pertence a X, ele serah da forma 3*(2m-1) + 2 = 6m - 1, com m > 0. De fato, podemos dizer que m > 1, pois se 5 pertencesse a X, entao:5^2 + 4 = 29 pertence a X ==>29^2 + 4 = 845 pertence a X ==>contradicao, pois 845 eh multiplo de 5.Por enquanto isso foi tudo que eu consegui e suspeito que tenha sido a parte mais facil. Enfim, jah eh um comeco![]s,Claudio. 

TRANSIRE SVVM PECTVS MVNDOQVE POTIRI
CONGREGATI EX TOTO ORBE MATHEMATICI OB SCRIPTA INSIGNIA TRIBVERE
Fields Medal(John Charles Fields)Yahoo! Messenger - Fale com seus amigos online. Instale agora!

Re: [obm-l] Teoria dos Numeros

2004-04-17 Por tôpico Claudio Buffara
Title: Re: [obm-l] Teoria dos Numeros



on 17.04.04 10:56, Johann Peter Gustav Lejeune Dirichlet at [EMAIL PROTECTED] wrote:

Seja X o conjunto dos primos tais que se a e b sao dois elementos dele entao ab+4 e a^2+4 tambem estao.Prove ou disprove: X e vazio


Inicio de solucao:
   
Suponhamos que X <> vazio e seja n um elemento de X.
Eh facil ver que n > 2.
Alem disso, n > 3, pois se 3 pertencesse a X, teriamos:
3^2 + 4 = 13 em X ==>
3*13 + 4 = 43 em X ==>
13^2 + 4 = 173 em X ==>
43*173 + 4 = 7443 em X ==>
contradicao, pois 7443 = 3*2481 eh composto.

Agora, um primo p > 3 eh da forma 3k + 1 ou 3k + 2.
Se p = 3k + 1 pertence a X, entao:
p^2 + 4   e   p*(p^2 + 4) + 4 = p^3 + 4p + 4 pertencem a X.
Mas p^3 + 4p + 4 = (p^3 - p) + 3*(p + 1) + (2p + 1). 
Alem disso:
p^3 - p eh multiplo de 3, pelo pequeno teorema de Fermat;
3*(p + 1) eh multiplo de 3 (obviamente);
2p + 1 = 2*(3k + 1) + 1 = 3*(2k + 1) eh multiplo de 3.
Ou seja, p*(p^2 + 4) + 4 eh multiplo de 3 e, portanto, composto ==>
contradicao ==>
nenhum primo da forma 3k + 1 pertence a X.

Em suma, se algum primo pertence a X, ele serah da forma 3k + 2.
Mais ainda: se k for par (por exemplo, k = 2m, com m > 0), teremos que:
3k + 2 = 3*(2m) + 2 = 2*(3m + 1) eh par e, portanto, nao pode pertencer a X.

Logo, se algum primo pertence a X, ele serah da forma 3*(2m-1) + 2 = 6m - 1, com m > 0. 

De fato, podemos dizer que m > 1, pois se 5 pertencesse a X, entao:
5^2 + 4 = 29 pertence a X ==>
29^2 + 4 = 845 pertence a X ==>
contradicao, pois 845 eh multiplo de 5.

Por enquanto isso foi tudo que eu consegui e suspeito que tenha sido a parte mais facil. Enfim, jah eh um comeco!


[]s,
Claudio. 






Re: [obm-l] teoria dos numeros

2004-01-21 Por tôpico Nicolau C. Saldanha
On Wed, Jan 21, 2004 at 04:46:28PM -0300, levi queiroz wrote:
> Pessoal da lista , eu estou enviando para de vocês quatro proposições minhas
> que eu mesmo demonstrei e no entanto eu não sei se constam dentro da Teoria
> dos Números. Gostaria da ajuda de vocês.
> 
> Proposição 1: Se  p >3 e p+2 são primos gêmeos então  p +1 = 6k, para algum k
> inteiro
...
> Proposição 2: Se p>3 e  p+2 são primos gêmeos então  p= 6.k +5, para algum k
> inteiro
...
> Proposição 3: Se  p>3 e 2.p +1 são primos então  p+1=6.n , para algum n
> inteiro.
...
> Proposição 4 : Se p>3 e 2p +1 são primos então  p= 6.k +5, para algum k
> inteiro.

Eu não entendi direito que tipo de ajuda você quer. As proposições estão
corretas, as demonstrações tanto quanto eu verifiquei também estão.
Este assunto é teoria dos números; é isso que você queria perguntar?
Ou talvez você estivesse perguntando se os resultados são conhecidos?
Sim, são bem conhecidos.

Não se sabe por outro lado se existem infinitos pares de primos gêmeos.
Se p é primo e 2p+1 também é primo, então p é chamado um primo
de Sophie Germain. Também não se sabe se existem infinitos primos
de Sophie Germain.

Para saber mais sobre números primos eu recomendo que você dê uma olhada em
http://www.utm.edu/research/primes
ou
http://primes.utm.edu

Para primos gêmeos veja
http://primes.utm.edu/top20/page.php?id=1

Tem uma curiosidade lá sobre o infame bug do pentium.

Para primos de Sophie Germain veja
http://primes.utm.edu/top20/page.php?id=2

[]s, N.
=
Instruções para entrar na lista, sair da lista e usar a lista em
http://www.mat.puc-rio.br/~nicolau/olimp/obm-l.html
=


Re: [obm-l] Teoria dos numeros

2003-09-15 Por tôpico Johann Peter Gustav Lejeune Dirichlet
Voce perde a generalidade sim, pois por exemplo
a=401 e b=5001 nao sao da forma descrita.
 --- Henrique_Patrício_Sant'Anna_Branco
<[EMAIL PROTECTED]> escreveu: > Prove as
seguintes afirmações:
> a) Se a é um inteiro ímpar, então 24 divide
> a*(a^2 - 1)
> b) Se a e b são inteiros impares, entao 8
> divide a^2 - b^2
> No caso do item b) pensei em considerar a =
> 4k-1 e b = 4k+1. Eu perco a
> generalidade se fizer algo assim?
> 
> Grato,
> Henrique.
> 
>
=
> Instruções para entrar na lista, sair da lista
> e usar a lista em
>
http://www.mat.puc-rio.br/~nicolau/olimp/obm-l.html
>
= 

___
Desafio AntiZona: participe do jogo de perguntas e respostas que vai
dar um Renault Clio, computadores, câmeras digitais, videogames e muito
mais! www.cade.com.br/antizona
=
Instruções para entrar na lista, sair da lista e usar a lista em
http://www.mat.puc-rio.br/~nicolau/olimp/obm-l.html
=


Re: [obm-l] Teoria dos numeros

2003-09-15 Por tôpico Marcelo Souza
a) a(a^2-1)
Se a e impar entao
a^2==1 mod 8
e como (a-1)a(a+1) são tres inteiros consecutivos, temos que 3 tb o divide, 
logo 24 divide o produto

b)
Mesmo esquema
a^2==1 mod 8
b^2==1 mod 8
a^2-b^2==0 mod 8

obs.: Considere a=8k+r onde 0=

e eleve ao quadrado para os casos impares (para nao fazer mta conta use ao 
inves de r=5, r=-3 (por ai), que fica bem resumido, dai vc observar que o 
quadrado de um numero inteiro impar e congruente a 1 mod 8).

[]'s, Marcelo.

From: "Henrique Patrício Sant'Anna Branco" <[EMAIL PROTECTED]>
Reply-To: [EMAIL PROTECTED]
To: "OBM" <[EMAIL PROTECTED]>
Subject: [obm-l] Teoria dos numeros
Date: Sun, 14 Sep 2003 20:37:26 -0300
Prove as seguintes afirmações:
a) Se a é um inteiro ímpar, então 24 divide a*(a^2 - 1)
b) Se a e b são inteiros impares, entao 8 divide a^2 - b^2
No caso do item b) pensei em considerar a = 4k-1 e b = 4k+1. Eu perco a
generalidade se fizer algo assim?
Grato,
Henrique.
=
Instruções para entrar na lista, sair da lista e usar a lista em
http://www.mat.puc-rio.br/~nicolau/olimp/obm-l.html
=
_
MSN Messenger: instale grátis e converse com seus amigos. 
http://messenger.msn.com.br

=
Instruções para entrar na lista, sair da lista e usar a lista em
http://www.mat.puc-rio.br/~nicolau/olimp/obm-l.html
=


Re: [obm-l] Teoria dos numeros

2003-09-14 Por tôpico Claudio Buffara
on 14.09.03 20:37, Henrique Patrício Sant'Anna Branco at
[EMAIL PROTECTED] wrote:

> Prove as seguintes afirmações:
> a) Se a é um inteiro ímpar, então 24 divide a*(a^2 - 1)
> b) Se a e b são inteiros impares, entao 8 divide a^2 - b^2
> No caso do item b) pensei em considerar a = 4k-1 e b = 4k+1. Eu perco a
> generalidade se fizer algo assim?
> 
Infelizmente voce perde, pois poderia ser a = 4k+1, por exemplo.

a) a impar ==>
a = 2k+1 para algum inteiro k ==>
a(a^2-1) = (2k+1)(4k^2+4k) = 4k(k+1)(2k+1)

Agora voce raciocina assim:
k e k+1 sao inteiros consecutivos ==>
um deles eh par ==>
2 divide k(k+1) ==>
8 divide 4k(k+1)(2k+1)  (*)

Se 3 divide k ou 3 divide k+1, entao 3 divide 4k(k+1)(2k+1) ==>
juntamente com (*) isso implica que 24 (=8*3) divide 4k(k+1)(2k+1)

Se 3 nao divide k nem k+1, entao k = 3m+1, para algum inteiro m ==>
2k+1 = 6m+3 ==>
3 divide 2k+1 ==>
3 divide 4k(k+1)(2k+1) ==>
juntamente com (*) isso implica que 24 divide 4k(k+1)(2k+1)

*

b) Na verdade, isso eh decorrencia do fato de que se a eh impar entao 8
divide a^2 - 1, pois a = 2m + 1 ==>
a^2 - 1 = 4m^2 + 4m + 1 - 1 = 4m(m+1)

Mas, como visto acima, 2 divide m(m+1) ==>
8 divide 4m(m+1) = a^2 - 1.


Um abraco,
Claudio.
a^2 - b^2 = 4m^2 + 4m - 4n^2 - 4n = 4[m(m+1) - n(n+1)]

=
Instruções para entrar na lista, sair da lista e usar a lista em
http://www.mat.puc-rio.br/~nicolau/olimp/obm-l.html
=


Re: [obm-l] Teoria dos numeros

2003-09-14 Por tôpico Murilo
a) a*(a^2 - 1) = a*(a-1)*(a+1)=(a+1)*a*(a-1)

Notamos que sao 3 numeros consecutivos, e seja a impar, a-1 e a+1 sao pares
q contem um multiplo de 2 e outro de 4, claramente. E em 3 numeros
consecutivos, a probabilidade de se encontrar um multiplo de 3 eh 100% logo
eh multiplo de 4*3*2=24

b) a^2 - b^2=(a+b)*(a-b)

Novamente, seja a e b dois numeros impares, (a+b) e (a-b) sao pares.
a=2*p+1
b=2*q+1

(a+b)*(a-b) = (2*p + 2*q + 2)*(2*p + 2*q) = 2*(p+q+1)*2*(p+q) =
2*2*(p+q+1)*(p+q)
como (p+q) e (p+q+1) sao consecutivos, um dos dois há de ser par logo
possuem o fator dois.
logo eh multiplo de 2*2*2 = 8

- Original Message -
From: "Henrique Patrício Sant'Anna Branco" <[EMAIL PROTECTED]>
To: "OBM" <[EMAIL PROTECTED]>
Sent: Sunday, September 14, 2003 8:37 PM
Subject: [obm-l] Teoria dos numeros


> Prove as seguintes afirmações:
> a) Se a é um inteiro ímpar, então 24 divide a*(a^2 - 1)
> b) Se a e b são inteiros impares, entao 8 divide a^2 - b^2
> No caso do item b) pensei em considerar a = 4k-1 e b = 4k+1. Eu perco a
> generalidade se fizer algo assim?
>
> Grato,
> Henrique.
>
> =
> Instruções para entrar na lista, sair da lista e usar a lista em
> http://www.mat.puc-rio.br/~nicolau/olimp/obm-l.html
> =
>

=
Instruções para entrar na lista, sair da lista e usar a lista em
http://www.mat.puc-rio.br/~nicolau/olimp/obm-l.html
=


Re: [obm-l] Teoria dos Numeros

2002-07-15 Por tôpico Nicolau C. Saldanha

On Sun, Jul 14, 2002 at 04:45:19PM -0300, adr.scr.m wrote:
> Determine todos inteiros positivos 
> x,y,z,tais  que z divide xy-1,x divide zy-1
> e y divide zx-1.

Este problema 'e bem legal. Vou pular umas linhas antes de dar a solu,c~ao
para que os outros tentem fazer sozinhos, vale a pena.

==







































==

Uma solu,c~ao trivial 'e x=y=z=1. Qualquer outra solu,c~ao pode ser tomada
da forma 1http://www.mat.puc-rio.br/~nicolau/olimp/obm-l.html
O administrador desta lista é <[EMAIL PROTECTED]>
=



Re: [obm-l] Teoria dos Numeros

2002-07-15 Por tôpico Nicolau C. Saldanha

On Sun, Jul 14, 2002 at 04:45:19PM -0300, adr.scr.m wrote:
> Alguem poderia fazer essas questoes para  
> mim ?
> 
> Determine todos os primos que sao a soma e  
> a diferenca de 2 primos.

5 = 3 + 2 = 7 - 2 'e o 'unico.
Basta observar que 'e indispens'avel usar o primo 2
e que o 'unico caso em que p-2, p e p+2 s~ao todos primos 'e para p=5
pois um dos tr^es sempre ser'a m'ultiplo de 3.

[]s, N.
=
Instruções para entrar na lista, sair da lista e usar a lista em
http://www.mat.puc-rio.br/~nicolau/olimp/obm-l.html
O administrador desta lista é <[EMAIL PROTECTED]>
=